Đến nội dung

Secrets In Inequalities VP nội dung

Có 298 mục bởi Secrets In Inequalities VP (Tìm giới hạn từ 26-05-2020)



Sắp theo                Sắp xếp  

#339335 Cho 3 số dương thỏa ab+bc+ca=3.CMR $\sum \frac{1}...

Đã gửi bởi Secrets In Inequalities VP on 23-07-2012 - 20:23 trong Bất đẳng thức và cực trị

Cho 3 số dương a, b, c thỏa mãn ab + bc + ca = 3. CMR : $\sum \frac{1}{1+a^2(b+c)}\leq \frac{1}{abc}$

Cách khác ;
Ta có : $ab+bc+ca\geq 3\sqrt[3]{(abc)^2}\Rightarrow abc\leq 1$
$\Rightarrow \sum \frac{1}{1+a^2(b+c)}\leq \sum \frac{1}{abc+a^2(b+c)}= \sum \frac{1}{a(bc+ac+ac)}= \sum \frac{1}{3a}$
$= \frac{1}{3a}+\frac{1}{3b}+\frac{1}{3c}= \frac{ab+bc+ca}{3abc}= \frac{1}{abc}$



#298632 $x+y+z\geq \frac{1}{\sqrt{x}}+\frac{1}{\sqrt{y}...

Đã gửi bởi Secrets In Inequalities VP on 08-02-2012 - 19:28 trong Bất đẳng thức và cực trị

cho x,y,z dương và x+y+z=3 crm $x+y+z\geq \frac{1}{\sqrt{x}}+\frac{1}{\sqrt{y}}+\frac{1}{\sqrt{z}}$

Bài này sai đề rồi .
Ta có : $ \frac{1}{\sqrt{x}}+\frac{1}{\sqrt{y}}+\frac{1}{\sqrt{z}}\geq \frac{9}{\sqrt{x}+\sqrt{y}+\sqrt{z}}\geq \frac{9}{\sqrt{3(x+y+z)}}= 3= x+y+z$



#353192 $$\sum \frac{1}{3+2(a^2-bc)}\geq...

Đã gửi bởi Secrets In Inequalities VP on 09-09-2012 - 17:18 trong Bất đẳng thức - Cực trị

Bài toán 2.
Ch0 $a,b,c,d>0$.Chứng minh:
$$\sqrt{\frac{ab+ac+ad+bc+bd+cd}{6}}\geq \sqrt[3]{\frac{abc+bcd+acd+abd}{3}}$$

Chuẩn hóa : $ab+ac+ad+bc+bd+cd= 6$
Bất đẳng thức trở thành $abc+bcd+cda+dab\leq 4$
WLOG assume $a\geq b\geq c\geq d$
Theo $Cauchy-Schwarz$
$VT^{2}\leq (ab+bc+cd+da)(abc^2+bcd^2+cda^2+dab^2)$
$= (ab+bc+cd+da)[ac(bc+da)+bd(ab+cd)]$
Do $a\geq b\geq c\geq d$ nên dễ thấy $ac\geq bd$ và
$(bc+da)-(ab+cd)= -(a-c)(b-d)\leq 0$ $\Rightarrow ab+cd\leq bc+da$
Thế nên áp dụng $Chebyshev$ ta được
$ac(bc+da)+bd(ab+cd)\leq \frac{(ac+bd)[(bc+da)+(ab+cd)]}{2}$
Từ hai dòng màu đỏ ta có được
$VT^2\leq \frac{(ab+bc+cd+da)^2(ac+bd)}{2}= \frac{(6-ac-bd)^2(2ac+2bd)}{4}$
Mà theo AM-GM $\frac{(6-ac-bd)^2(2ac+2bd)}{4}\leq \frac{1}{4}.[\frac{2(6-ac-bd)+2(ac+bd)}{3}]^3= 16$
Suy ra $VT^2\leq 16\Rightarrow VT\leq 4$



#364522 $\frac{a^{2}}{3a^{2}+(b+c)^...

Đã gửi bởi Secrets In Inequalities VP on 24-10-2012 - 19:37 trong Bất đẳng thức và cực trị

Cho các số thực không âm a,b,c thoả mãn a+b+c>0.
Chứng minh rằng:
$\frac{a^{2}}{3a^{2}+(b+c)^{2}}+\frac{b^{2}}{3b^{2}+(a+c)^{2}}+\frac{c^{2}}{3c^{2}+(a+b)^{2}}\leq \frac{1}{2}$
---------------------------
Không đặt tiêu đề quá dài bạn nhé :)

Ta sẽ CM : $\frac{a^2}{3a^2+(b+c)^2}\leq \frac{a}{2(a+b+c)}$
$\Leftrightarrow 2a(a+b+c)\leq 3a^2+(b+c)^2$
$\Leftrightarrow 2a^2+2ab+2ac\leq 3a^2+b^2+c^2+2bc$
$\Leftrightarrow a^2+b^2+c^2+2bc-2ab-2ac\geq 0$
$\Leftrightarrow (b+c-a)^2\geq 0$.Luôn đúng .
Tương tư có 2 bđt nua rồi cộng vào $\Rightarrow Q.E.D$



#317735 Tìm GTLN của $$A=\sum \frac{x}{x^2+yz}$$

Đã gửi bởi Secrets In Inequalities VP on 18-05-2012 - 21:47 trong Bất đẳng thức và cực trị

Tìm GTLN của biếu thức biết $x,y,z$ là các số thực dương và $x^2+y^2+z^2=xyz$
$$A=\frac{x}{x^2+yz}+\frac{y}{y^2+zx}+\frac{z}{z^2+xy}$$

GT $\Leftrightarrow \frac{x}{yz}+\frac{y}{zx}+\frac{z}{xy}= 1$
$A= \sum \frac{x}{x^{2}+yz}\leq\sum \frac{x}{4}.(\frac{1}{x^{2}}+\frac{1}{yz})= \sum \frac{1}{4}.(\frac{1}{x}+\frac{x}{yz})$
$= \frac{1}{4}.(\frac{1}{x}+\frac{1}{y}+\frac{1}{z}+\frac{x}{yz}+\frac{y}{zx}+\frac{z}{xy})= \frac{1}{4}.(\frac{1}{x}+\frac{1}{y}+\frac{1}{z}+1)$
Theo AM-GM : $\frac{x}{yz}+\frac{y}{zx}\geq \frac{2}{z}$
$\frac{y}{zx}+\frac{z}{xy}\geq \frac{2}{x}$
$\frac{z}{xy}+\frac{x}{yz}\geq \frac{2}{y}$
$\Rightarrow \frac{1}{x}+\frac{1}{y}+\frac{1}{z}\leq \frac{x}{yz}+\frac{y}{zx}+\frac{z}{xy}= 1$
$\Rightarrow A\leq \frac{1}{4}.(1+1)= \frac{1}{2}$
Dấu "=" khi $x=y=z=3$ .



#385745 Chứng minh rằng tồn tại $1$ số chia hết cho $2^k$ mà khi...

Đã gửi bởi Secrets In Inequalities VP on 11-01-2013 - 22:09 trong Số học

Bài toán : Chứng minh rằng tồn tại $1$ số chia hết cho $2^k$ mà khi viết nó trong hệ thập phân không chứa số $0$ nào với mọi $k$ nguyên dương.

Bài này cho vào box Số học thíc hợp hơn !
Ta sẽ cm bằng qui nạp :
+ $k=1$ ta có số $2$ thỏa mãn.
+ Giả sử kết luận của bài toán đúng tới $k$, ta cm nó đúng với $k+1$.
Theo gtqn tồn tại số $A\vdots 2^{k}$ có $k$ c/số tm đề bài.
Nếu $A\vdots 2^{k+1}$ thì thêm c/số $2$ vào trc $A$ ta đc số $2.10^{k}+A= 2^{k+1}.5^{k}+A\vdots 2^{k+1}$ và số này thỏa đề
Nếu $A\vdots 2^{k};A\not\vdots 2^{k+1}$ thì viết $A= 2^{k}.p$ trong đó $p$ lẻ.
Xét số có được sau khi thêm c/số $1$ vào trc số $A$ : $10^{k}+A= 2^{k}.(5^{k}+p)\vdots 2^{k+1}$ do $p$ lẻ và do đó số này thỏa đề.
Vậy theo nguyên lí qui nạp ta có $Q.E.D$



#398581 $\begin{matrix} f:\mathbb{N}\rightarrow \mathbb{N}...

Đã gửi bởi Secrets In Inequalities VP on 20-02-2013 - 18:22 trong Phương trình hàm

$\begin{matrix} f:\mathbb{N}\rightarrow \mathbb{N}\\ mf(n)+nf(m)=(m+n)f(m^{2}+n^{2})\forall m,n\epsilon \mathbb{N} \end{matrix}$

Ta thấy ngay hàm hằng $f(x)=c$ thỏa mãn.Giả sử $f$ k là hàm hằng.
Chọn $m$ và $n$ là $2$ số sao cho $f(m)-f(n)>0$ và nhỏ nhất.
$\Rightarrow f(n)= \frac{nf(n)+mf(n)}{m+n}< \frac{nf(m)+mf(n)}{m+n}< \frac{nf(m)+mf(m)}{m+n}= f(m)$
$\Rightarrow f(n)< f(m^2+n^2)< f(m)$
$\Rightarrow 0<f(m^2+n^2)-f(n)< f(m)-f(n)$ .Vô lí theo cách chọn $m,n$



#448240 Chứng minh rằng : $m\vdots p$

Đã gửi bởi Secrets In Inequalities VP on 06-09-2013 - 18:38 trong Số học

CHo $p$ là số nguyên tố lẻ.Kí hiệu : ${S_a}= a+\frac{a^{2}}{2}+...+\frac{a^{p-1}}{p-1}$.

Giả sử ${S_3}+{S_4}-3{S_2}=\frac{m}{n}$. Chứng minh rằng : $m\vdots p$




#381865 chứng minh rằng hoặc p2 | a + b, hoặc p3 | a3 + b3

Đã gửi bởi Secrets In Inequalities VP on 30-12-2012 - 08:16 trong Số học

Cho p > 3 là số nguyên tố. Giả sử a, b là các số nguyên sao cho p | a + b và p2 | a3 + b3. Hãy chứng minh rằng hoặc p2 | a + b, hoặc p3 | a3 + b3

Vì $p^{2} | a ^{3}+ b^{3}= (a+b)(a^2-ab+b^2)$ nên chỉ xảy ra
+ $p^{2} | (a+b)$ ta có đpcm
+ $p | a^2-ab+b^2$ .Mà $p | a+b\Rightarrow p | (a+b)^2= a^2+2ab+b^2$
$\Rightarrow p | 3ab$ .Mà $p>3$ $\Rightarrow (p,3)=1\Rightarrow p | ab$
Lại có $p | a+b,p \in \mathbb{P}$ nên $p | a$ và $p | b$
$\Rightarrow p^{3} | a^{3}, p^{3} | b^{3}\Rightarrow p^{3} | a^{3}+b^3$



#369746 $S = C_{2n + 1}^0{.2^{2n}} + C_{2n +...

Đã gửi bởi Secrets In Inequalities VP on 15-11-2012 - 21:56 trong Số học

Đặt $a=1+\sqrt{3},b= 1-\sqrt{3}\Rightarrow ab=-2;\frac{a^2}{2}= 2+\sqrt{3};\frac{b^2}{2}= 2-\sqrt{3}$
${P_n}= \frac{1}{2}(a^{2n+1}+b^{2n+1})$
Dùng khai triển Newton cho $(1+\sqrt{3})^{2n+1},(1-\sqrt{3})^{2n+1}$ta suy ra đc ${P_n}= \sum_{k=0}^{n}\binom{2k}{2n+1}.3^k$ là số nguyên .
Lại dùng Newton cho $(2+\sqrt{3})^{2n+1},(2-\sqrt{3})^{2n+1}$ ta đc:
${S_n}= \frac{(\frac{a^2}{2})^{2n+1}+(\frac{b^2}{2})^{2n+1}}{4}= \frac{a^{4n+2}+b^{4n+2}}{2^{2n+3}}= \frac{a^{4n+2}+2(ab)^{2n+1}+b^{4n+2}}{2^{2n+3}}+\frac{1}{2}= \frac{{P_n}^{2}}{2^{2n+1}}+\frac{1}{2}$
Nhân chéo lên
$\Rightarrow {P_n}^{2}\vdots 2^{2n},{P_n}^{2}\not\vdots 2^{2n+1}\Rightarrow {P_n}^{2}= m2^{2n}$ ( m lẻ )
Mà ${P_n}^{2}$ và $2^{2n}$ chính phuong nên $m= (2p+1)^{2}$
$\Rightarrow {P_n}^{2}= (2p+1)^22^{2n}$
$\Rightarrow {P_n}= (2p+1)2^{^{n}}\Rightarrow\frac{{P_n}-2^n}{2^{n+1}}= p$
Mà dễ thấy :
${S_n}= \frac{{P_n}^{2}}{2^{2n+1}}+\frac{1}{2}= (\frac{{P_n}-2^n}{2^{n+1}})^{2}+(\frac{{P_n}+2^n}{2^{n+1}})^{2}= p^{2}+(p+1)^{2}$
Vậy ta có $Q.E.D$



#362816 Chứng minh rằng với mỗi số nguyên tố p có vô số dạng $2^{n}-n...

Đã gửi bởi Secrets In Inequalities VP on 18-10-2012 - 19:37 trong Số học

Chứng minh rằng với mỗi số nguyên tố p có vô số dạng $2^{n}-n$ chia hết cho p.

+ Nếu $p=2$ thì n cứ chẵn là thỏa mãn .
+ Nếu $p> 2$ $\Rightarrow p$ lẻ .
Theo Fermat nhỏ $2^{p-1}-1\vdots p\rightarrow 2^{m(p-1)}-1\vdots p$
Chọn $n= m(p-1)$ trong đó $m\equiv -1(mod p)$ suy ra $n=1(modp)$
$\rightarrow 2^n-n\equiv 2^n-1\equiv 0(modp)$ $\rightarrow 2^n-1\vdots p$
Vì có vô số số $m\equiv -1(modp)$ nên có vô số thỏa mãn đề bài .



#375640 Chứng minh rằng phương trình $x^{4}+3y^{4}=z^{4...

Đã gửi bởi Secrets In Inequalities VP on 06-12-2012 - 20:38 trong Số học

Chứng minh rằng phương trình $x^{4}+3y^{4}=z^{4}$ có vô số nghiệm nguyên

Ta thấy : $k^{4}+3.0^{4}= k^4$
Vậy pt có vô số nghiệm nguyên $(x,y,z)=(k,0,k)$ với $k\in\mathbb{Z}$



#375309 Xác định giá trị $p,q$ để $S$ là một số nguyên tố

Đã gửi bởi Secrets In Inequalities VP on 05-12-2012 - 15:19 trong Số học

Ta có nhận xét là nếu $a \not\in\mathbb{Z}$ thì $[a]+[-a]= -1$
Xét : $[\frac{kq}{p}]+[\frac{(p-k)q}{q}]= [\frac{kq}{p}]+[q-\frac{kp}{q}]= q+[\frac{kq}{p}]+[-\frac{kq}{p}]$ $= q-1$
( vì do $(p,q)=1$ nên $\frac{kp}{q}\not\in\mathbb{Z}$ nên áp dụng NX trên thì ta có đc điều này )
Tương tự suy ra : $S= \frac{(p-1)(q-1)}{2}$
Dễ rồi !



#375996 Chứng minh rằng tồn tại vô số n để $a_n$ chia hết cho 7

Đã gửi bởi Secrets In Inequalities VP on 08-12-2012 - 14:28 trong Dãy số - Giới hạn

Cho $\large{(a_n)\begin{cases}a_0=1\\a_n=a_{n-1}+a_{[\frac{n}{2}]}\end{cases}}$
Chứng minh rằng tồn tại vô số n để $a_n$ chia hết cho 7

Giả sử chỉ có hữu hạn số chia hết cho $7$ và $a_k$ là số cuối cùng trong các số ấy
Theo ct xác định dãy ta có : $a_{2k+1}= a_{2k}+a_k;a_{2k}= a_{2k-1}+a_k$
$\Rightarrow a_{2k-1}\equiv a_{2k}\equiv a_{2k+1}\equiv b \pmod 7$
Ta có : $a_{4k-3}\equiv a_{4k-3}+0.b \pmod 7$
$a_{4k-2}= a_{4k-3}+a_{2k-1}\equiv a_{4k-3}+1.b \pmod 7$
$a_{4k-1}= a_{4k-2}+a_{2k-1}\equiv a_{4k-3}+2.b \pmod 7$
$a_{4k}= a_{4k-1}+a_{2k}\equiv a_{4k-3}+3.b \pmod 7$
$a_{4k+1}= a_{4k}+a_{2k}\equiv a_{4k-3}+4.b \pmod 7$
$a_{4k+2}= a_{4k+1}+a_{2k+1} \equiv a_{4k-3}+5.b \pmod 7$
$a_{4k+3}= a_{4k+2}+a_{2k+1} \equiv a_{4k-3}+6.b \pmod 7$
Do $\gcd (b,7)=1$ nên các số $a_{4k-3}+ib$ với $i$ chạy từ $0$ đến $6$ lập thành 1 HĐĐ modulo 7.
Nên trong $7$ số $a_{4k-3}+ib$ phải có một số chia hết cho $7$, mà số này lại lớn hơn $a_k$: mâu thuẫn vs điều giả sử
Vậy giả sử là sai và ta có $Q.E.D$

p/s :Ai sửa hộ mình cái Latex vs .Thanks trước !

Perfectstrong: Đề nghị em nên học một lớp bổ túc về latex gấp :P



#343716 Chứng minh $(a+b)^2+(a+b+4c)^2\ge \frac{100abc}...

Đã gửi bởi Secrets In Inequalities VP on 05-08-2012 - 18:27 trong Bất đẳng thức và cực trị

(HongKong TST 2001) Cho $a,b,c$ thực dương. Chứng minh $(a+b)^2+(a+b+4c)^2\ge \frac{100abc}{a+b+c}$

BĐT $\Leftrightarrow [(a+b)^2+(a+b+4c)^2](a+b+c)\geq 100abc$
Mà $(a+b)^2+(a+b+4c)^2= (a+b)^2+[(a+b)+4c]^2\geq 4ab+16c(a+b)=4(ab+4bc+4ac)$
Nên ta chỉ cần CM : $2(a+b+c)(ab+4bc+4ca)\geq 50abc$
Đúng vì :
$2(a+b+c)=a+a+b+b+2c\geq 5\sqrt[5]{2a^2b^2c}$

$ab+4bc+4ca= ab+2bc+2bc+2ca+2ca\geq 5\sqrt[5]{16a^3b^3c^4}$
Nhân 2 vế các BĐT trên ta đk ngay Đ.P.C.M .



#308232 $ abc+2\sum a^{2}+8\geq 5\sum a$

Đã gửi bởi Secrets In Inequalities VP on 04-04-2012 - 22:04 trong Bất đẳng thức và cực trị

Cho $ a,b,c\geq 1$ . CMR :
$ abc+2(a^{2}+b^{2}+c^{2})+8\geq 5(a+b+c)$.
-----------------------------------------------------------------------------------
Bài này thục ra ĐK là $ a,b,c\geq 0$ thui , nhg nếu nhu thế thj` sẽ khó hon .
Ai có time thì CM thủ nhé !



#304557 $$\sum{\dfrac{a + b}{a^2 + b^2} }\ge \dfrac{3...

Đã gửi bởi Secrets In Inequalities VP on 16-03-2012 - 15:17 trong Bất đẳng thức và cực trị

Chứng minh rằng, với mọi $a, b, c$ dương, ta có :
$$\dfrac{a + b}{a^2 + b^2} + \dfrac{b + c}{b^2 + c^2} + \dfrac{c + a}{c^2 + a^2} \ge \dfrac{3\left (a + b + c\right )}{a^2 + b^2 + c^2}$$




BĐT : $ \Leftrightarrow \sum (\frac{a+b}{a^{2}+b^{2}}-\frac{a+b+c}{a^{2}+b^{2}+c^{2}})\geq 0$
$ \Leftrightarrow \sum \frac{(a+b)(a^{2}+b^{2}+c^{2})-(a^{2}+b^{2})(a+b+c)}{(a^{2}+b^{2})(a^{2}+b^{2}+c^{2})}\geq 0$
$ \Leftrightarrow \sum \frac{c^{2}(a+b)-c(a^{2}+b^{2})}{(a^{2}+b^{2})(a^{2}+b^{2}+c^{2})}\geq 0$
$ \Leftrightarrow \sum \frac{ca(c-a)+cb(c-b)}{a^{2}+b^{2}}\geq 0$
$ \Leftrightarrow \sum (\frac{ca(c-a)}{a^{2}+b^{2}}+\frac{ac(a-c)}{b^{2}+c^{2}})\geq 0$
$ \Leftrightarrow \sum ac(a-c)(\frac{1}{b^{2}+c^{2}}-\frac{1}{a^{2}+b^{2}})\geq 0$
$ \Leftrightarrow \sum ac(a-c)\frac{a^{2}-c^{2}}{(a^{2}+b^{2})(b^{2}+c^{2})}\geq 0$
$ \Leftrightarrow \sum \frac{ac(a-c)^{2}(a+c)}{(a^{2}+b^{2})(b^{2}+c^{2})}\geq 0$ .
( luôn đúng ).
Dấu "=" xảy ra $ \Leftrightarrow a=b=c$
.



#304410 $$3\left (x^2y + y^2z + z^2x\right )\left (xy^2 + yz...

Đã gửi bởi Secrets In Inequalities VP on 15-03-2012 - 17:15 trong Bất đẳng thức và cực trị

BĐT :
$\Leftrightarrow 3(\frac{x}{z}+\frac{y}{x}+\frac{z}{y})(xy^{2}+yz^{2}+zx^{2})\geq (x+y+z)^{3}$
$\Leftrightarrow (1+1+1)(\frac{x}{z}+\frac{y}{x}+\frac{z}{y})(zx^{2}+xy^{2}+yz^{2})\geq (x+y+z)^{3}$
(luôn đúng theo BĐT Holder)
Dấu "=" xảy ra khi và chỉ khi x = y = z .



#299401 tìm các phân số tối giản

Đã gửi bởi Secrets In Inequalities VP on 14-02-2012 - 18:27 trong Số học

Ta có : $ 60= 2^{2}.3.5= 4.3.5$
Ta thấy : tử số ko thể chỉ có 1 thùa số 2 vì nếu chỉ có 1 thùa số 2 thì khi đó mẫu cũng chúa thùa số 2 thì phân số ko tối giản .Do đó , tử sẽ chúa cả 2 thùa số 2 . Suy ra , tử số sẽ là các số 1,3,4,5 ( vì phân số bé hơn 1 ) .Vậy có 4 phân số thỏa mãn đề bài .



#308309 Trong mặt phẳng cho 8045 điểm

Đã gửi bởi Secrets In Inequalities VP on 05-04-2012 - 11:43 trong Hình học

Bài 5.2:
Gọi X là tập hợp các điểm đã cho trước.
Trong các tam giác có 3 đỉnh thuộc tập X, ta chọn $\vartriangle ABC$ là tam giác có diện tích lớn nhất.
Các đường thẳng qua A,B,C thứ tự song song với BC,CA,AB đôi một cắt nhau tại D,E,F như hình vẽ.
Ta chứng minh mọi điểm của tập X đều nằm trong $\vartriangle DFE$, kể cả trên cạnh.
Giả sử, có 1 điểm M thuộc tập X sao cho M nằm ngoài $\vartriangle DFE$. Không mất tính tổng quát, giả sử vị trí của M nằm như hình vẽ.
Hình đã gửi
Vẽ MA cắt FD tại H.
$S_{MAC}>S_{CAH}=S_{CAB}$: trái với cách chọn $\vartriangle ABC$ ban đầu.
Vậy ta có mọi phần tử của X đều nằm trong $\vartriangle DFE$ nên chúng nằm trong 1 trong 4 tam giác: ABC,ABF,ACE,BCD.
Theo nguyên lý Dirichlet, tồn tại 1 tam giác chứa 2012 điểm.
Lại có $S_{ABC}=S_{ABF}=S_{ACE}=S_{BCD}<1$ nên ta có đpcm.




#331119 CMR: $x^{3}+y^{3}\leq x^{2}+y^{2}\leq x+y\leq 2$

Đã gửi bởi Secrets In Inequalities VP on 02-07-2012 - 12:03 trong Bất đẳng thức và cực trị

Chứng minh BĐT 1 :
$x^{2}+y^{3}+y^{2}\geq x^{3}+y^{4}+y^{2}\geq x^{3}+2y^{3}\Rightarrow x^{2}+y^{2}\geq x^{3}+y^{3}$
Chứng minh BĐT 2 :
$x^{3}+x+y^{3}+y\geq 2x^{2}+2y^{2}\geq x^{3}+y^{3}+x^{2}+y^{2}\Rightarrow x+y\geq x^{2}+y^{2}$
Chứng minh BĐT 3 :
$(x+y)^{2}\leq 2(x^{2}+y^{2})\leq 2(x+y)\Rightarrow x+y\leq 2$
Cm hoàn tất.
Dấu "=" xảy ra o các BĐT khi và chỉ khi x=y=1



#334669 BĐT thi học sinh giỏi tỉnh lớp 9 năm 2011-2012

Đã gửi bởi Secrets In Inequalities VP on 12-07-2012 - 07:21 trong Bất đẳng thức và cực trị

Cho a,b,c,d là các số thực thỏa mãn điều kiện:
abc+bcd+cda+dab=a+b+c+d+$\sqrt{2012}$
Chứng minh rằng: (a2+1)(b2+1)(c2+1)(d2+1)\geq or \ge2012


Ta có:
$2012=( abc+bcd+cda+dab-a-b-c-d)^2 =( (ab - 1)(c + d) + (cd - 1)( a + b))$
$\leq ((ab -1)^2 +(a + b)^2)((cd -1)^2 +(c + d)^2)$
$=(a^2b^2 +a^2 +b^2+1)(c^2d^2 +c^2 +d^2 +1) =(a^2 +1)(b^2 +1)(c^2 +1)(d^2 +1)$
Đ.P.C.M.



#333909 $$\dfrac{1}{x_1y_1-z_1^2}+\dfrac{1}{x_2y_2-z_2^2}\ge...

Đã gửi bởi Secrets In Inequalities VP on 10-07-2012 - 08:42 trong Bất đẳng thức và cực trị

Bài toán
Cho các số dương $x_1, x_2$ và các số thực $y_1, y_2, z_1, z_2$ và $x_1y_1> z_1^2, x_2y_2> z_2^2$. Chứng minh rằng :
$$\dfrac{1}{x_1y_1-z_1^2}+\dfrac{1}{x_2y_2-z_2^2}\ge \dfrac{8}{(x_1+x_2)(y_1+y_2)-(z_1+z_2)^2}$$
Yêu cầu : Không được sử dụng những tài liệu liên quan ! Tự lực là tốt nhất .

Theo Cauchy-Schwarz :
$(z_1+z_2)^{2}= (\sqrt{x_1}.\frac{z_1}{\sqrt{x_1}}+\sqrt{x_2}.\frac{z_2}{\sqrt{x_2}})^2\leq (x_1+x_2)(\frac{z_1^2}{x_1}+\frac{z_2^2}{x_2})$
$\Rightarrow (x_1+x_2)(y_1+y_2)-(z_1+z_2)^2\geq (x_1+x_2)(y_1-\frac{z_1^2}{x_1}+y_2-\frac{z_2^2}{x_2})$
$\geq 2\sqrt{x_1x_2}.2\sqrt{(y_1-\frac{z_1^2}{x_1})(y_2-\frac{z_2^2}{x_2})}= 4\sqrt{(x_1y_1-z_1^2)(x_2y_2-z_2^2)}$
Do đó Áp dụng AM-GM :
$VT\geq \frac{2}{\sqrt{(x_1y_1-z_1^2)(x_2y_2-z_2^2)}}\geq \frac{8}{(x_1+x_2)(y_1+y_2)-(z_1+z_2)^2}$



#332452 cho a,b,c>0 có tổng bằng 1. CMR $\sum \frac{ab}{\sqrt...

Đã gửi bởi Secrets In Inequalities VP on 06-07-2012 - 10:33 trong Bất đẳng thức và cực trị

Theo BĐT Cauchy-schwarz ta có : $\sum \frac{ab}{\sqrt{ab+bc}}= \sum \frac{a\sqrt{b}}{\sqrt{c+a}}\leq \sum \frac{a\sqrt{2b}}{\sqrt{c}+\sqrt{a}}$
Ta cần CM : $\sum \frac{a\sqrt{2b}}{\sqrt{c}+\sqrt{a}}\leq 1$
Đặt $\sqrt{a}= x,\sqrt{b}= y,\sqrt{c}= z$
BĐT $\Leftrightarrow \sum \frac{2x^{2}y}{z+x}\leq 1= \sum x^{2}$
$\Leftrightarrow \sum x^{2}+\sum (2xy-\frac{2x^{2}y}{z+x})\geq 2\sum xy$$\Leftrightarrow \sum x^{2}+2xyz\sum \frac{1}{z+x}\geq 2\sum xy$
Mà $2xyz\sum \frac{1}{z+x}\geq \frac{9xyz}{x+y+z}$ nên BĐt cần CM đc đua về :
$\sum x^{2}+\frac{9xyz}{x+y+z}\geq 2\sum xy$
$\Leftrightarrow (x^{2}+y^{2}+z^{2})(x+y+z)+9xyz\geq 2(xy+yz+zx)(x+y+z)$
$\Leftrightarrow x^{3}+y^{3}+z^{3}+3xyz\geq ab(a+b)+bc(b+c)+ca(c+a)$
( Đúng theo Schur )
Đ.P.C.M OK



#354131 $t\vdots n$

Đã gửi bởi Secrets In Inequalities VP on 14-09-2012 - 19:48 trong Số học

Cho $n,k$ là các số tự nhiên thỏa mãn n không chia hết cho 3 và $k\geq n$.
Chứng minh rằng luôn tồn tại số nguyên dương t sao cho $t\vdots n$ và $S(t)=k$
$S(t)$ là tổng các chữ số của t